Math 73/103
Measure Theory and Complex Analysis
Last updated November 20, 2020 12:44:48 EST

General Information HW Assignments Canvas Page Lecture Videos


Announcements:


Homework Assigments

Assignments Made on:
Monday September 14:
  • Homework 0: Please fill out this survey entitled homework 0. Then submit your PDF responses via gradescope. Here are their instuctions for uploading PDFs: https://gradescope-static-assets.s3.amazonaws.com/help/submitting_hw_guide.pdf. This annoying exercise will make it less stressful when you need a upload an actual assignment. Really the only tricky bit here is figuring out how to assign page numbers to your answers so that I can more easily find them when I'm grading. I really hate grading, so you want to make me happy when I grade. Also, I hope no one is worried about how this assignment will effect their grade.
  • Work:
    1. Let $x$ and $y$ be complex sequences and let $xy$ be the sequence whose $n^{\rm th}$ term is $x_ny_n$. Assume $1< p < \infty$ and let $q$ be such that $\frac1p+\frac1q=1$. (We call $q$ the conjugate exponent to $p$.) Note that we allow $\|x\|_p=\infty$ if the sequence is not $p$-summable. Establish Holder's Inequality: $$\|xy\|_1\le\|x\|_p\|y\|_q$$ with the convention that $0\cdot \infty=0$. I suggest the following:
      1. Show that if $a,b\ge0$ and $0<\lambda<1$, then $a^\lambda b^{1-\lambda} \le \lambda a + (1-\lambda) b$. (Hint: reduce to the case $b>0$. Then let $t=a/b$ and use calculus to prove that $t^\lambda -\lambda t\le 1-\lambda$.)
      2. Use the observation that the norms are homogeneous to reduce to the case that $\|x\|_p=1=\|y\|_q$. (First take care of the cases where the norms are zero or infinite.)
      3. Now use part 1, with $a=|x_n|^p$, $b=|y_n|^q$ and $\lambda=\frac1p$.
      (Just for the record, if we agree to call $\infty$ the conjugate exponent to $p=1$, then it easy to see that we have $\|xy\|_1\le \|x\|_1\|y\|_\infty$ in this case as well.)
    2. Prove Minkowski's equality for $1\le p\le \infty$: namely that $\|x+y\|_p \le \|x\|_p+\|y\|_p$ for all $x\in \ell^p$. I suggest first reducing to the case $1< p <\infty$ and then applying Holder to $|x+y|^p\le (|x|+|y|)|x+y|^{p-1}$ where $|x|^r$ denotes the sequence with $n$th term $|x_n|^r$. (Observe that if $q$ is the conjugate exponent to $p$, then $(p-1)q=p$.)
    3. We want to show that the metrics induced on $\mathbf {R}^n$ by the $p$-norms are all strongly equivalent for $1\le p \le\infty$.
      • Observe that it will suffice to show that given $1\le p\le\infty$ there are $c,d>0$ such that $$c\|\mathbf x\|_2\le \|\mathbf x\|_p \le d\|\mathbf x\|_2\tag{*}$$ for all $\mathbf x\in\mathbf{R}^n$.
      • Prove (*) using the fact that a continuous function on a closed bounded subset $C$ of $\mathbf{R}^n$ attains its maximum and minimum on $C$. (Why is "attains" important above?)
Wednesday September 16:
    Work:
    1. Let $E$ be a subset of a metric space $X$. We say that $x$ is a limit point of $E$ if there is a sequence $(x_n)\subset E$ such that $x=\lim_{n\to\infty} x_n$. Show that $E$ is closed if and only if $E$ contains all its limit points.
    2. State and prove a result characterizing open sets in a metric space interms of sequences (as we did for closed sets in the previous problem). The following terminology might be useful. If $U$ is a subset of a metric space $X$, then a sequence $(x_n)\subset X$ is eventually in $E$ if there is a $N$ such that $n\ge N$ implies $x_n\in E$.
    3. Let $\rho$ and $\sigma$ be metrics on $X$. Show that $\rho$ and $\sigma$ are equivalent if and only if they have the same convergent sequences. That is, show that $x_n\to x$ with respect to $\rho$ if and only if $x_n\to x$ with respect to $\sigma$.
    4. Show that a Cauchy sequence in a metric space with a convergent subsequence is necessarily convergent.
    5. Let $X$ be a metric space. Prove that the uniform limit of continuous functions $f_n:X\to \mathbf C$ is continuous.
    6. Let $X$ be a metric space. Recall that we say $f:X\to \mathbf C$ is bounded if $\|f\|_\infty<\infty$. A sequence $(f_n)$ of functions $f_n:X\to\mathbf C$ is uniformly bounded if there is a $M$ such that $\|f_n\|_\infty\le M$ for all $n$. Also, $(f_n)$ is called uniformly Cauchy if for all $\epsilon>$ there is $N$ such that $n,m\ge N$ implies $|f_n(x)-f_m(x)|<\epsilon$ for all $x\in X$. Show that a uniformly Cauchy sequence $(f_n)$ of bounded functions is uniformly bounded. In particular, $(f_n)$ converges to a bounded function.
Friday September 18:
  • Work:
    1. Let $(X,\rho)$ be a metric space. If $A\subset X$, then define $\rho(x,A)=\inf\{\, \rho(x,y):y\in A\,\}$.
      • Show that $\rho(x,A)=0$ if and only if $x\in \overline A$.
      • Show that $x\mapsto \rho(x,A)$ is continuous.
      • Show that if $A$ and $B$ are disjoint nonempty closed subsets of $X$, then there is a $f\in C_b(X)$ such that (i) $0\le f(x)\le 1$ for all $x$, (ii) $f(x)=1$ if and only if $x\in A$, and (iii) $f(x)=0$ if and only if $x\in B$. (Hint: try $\rho(x,B)/(\rho(x,A)+\rho(x,B))$.)
  • Turn in: Problems 1-10 will be due Wednesday (September 23rd).
!-- Friday's assignment --> !-- Friday's assignment --> !-- Friday's assignment -->
Monday September 21:
  • Work:
    1. Show that $E\subset X$ is totally bounded if and only if there is an $\epsilon$-net for $E$ for all $\epsilon >0$.
    2. Suppose that $(X,\rho)$ is compact and that $f:(X,\rho)\to (Y,\sigma)$ is continuous. Show that $f(X)$ is compact in $Y$.
    3. Let $X=(0,1)$. For each $x\in X$, let $B_{\delta_x}(x)=\{\,y\in (0,1):|x-y|< \delta\,\}$ be such that $y\in B_{\delta_{x}}(x)$ implies $\bigl|\frac1x-\frac1y\bigr|<1$. Show that the cover $$(0,1)=\bigcup_{x\in(0,1)} B_{\delta_x}(x)$$ has no Lebesgue number.
    4. Show that a compact metric space has a countable dense subset. (Actually, it is enough for the space to be totally bounded.)
Wednesday:
  • Work:
    1. Let $\mathcal F$ be the family of functions $f_n(x)=x^n$ on $X=[0,1]$. Show that $\mathcal F$ is equicontinuous at each $x\in [0,1)$. (Luke, invoke the force in the form of the Mean Value Theorem.)
    2. Show that an equicontinuous family of functions on a compact metric space is uniformly equicontinuous as stated in lecture. (Some texts do not define equicontinuous at a point. Instead, whether $X$ is compact or not, equicontinuity is what we have called uniformly equicontinuity. Fortunately, there is no distinction for compact spaces.)
    3. Show that a compact subspace of a metric space must be closed. Conversely, show that a closed subspace of a compact metric space is compact.
Friday:
  • Homework Solutions: Here are selected solutions for the homework. (Last modified December 31, 1969)
  • Work:
    1. Let $K$ be a compact subset of a metric space $X$ and let $K \subset U$ with $U$ open. Show that there is an open set $V$ such that $K\subset V\subset \overline V\subset U$. (Consider homework problem #10.)
    2. (In this problem, we will assume that if $(X,\rho)$ and $(Y,\sigma)$ are metric spaces then so is $(X\times Y,\delta)$ where $\delta((x,y),(x',y'))=\rho(x,x')+\sigma(y,y')$. You can also assume that with respect to this product metric, $(x_n,y_n)\to (x,y)$ if and only if $x_n\to x$ and $y_n\to y$. In particular, if $(X,\rho)$ and $(Y,\sigma)$ are complete, so is $(X\times Y,\delta)$.) Let $U$ be a nonempty open subset of a complete metric space $(X,\rho)$. Show that $U$ admits a complete metric which is equivalent to that inherited from $X$. I suggest the following.
      • It suffices to find a homeomorphism $\phi:(U,\rho)\to (Y,\sigma)$ where $(Y,\sigma)$ is complete. (Recall that $\phi$ is a homeomorphism if $\phi$ is continuous, one-to-one and onto, and such that $\phi^{-1}$ is also continuous.)
      • Let $A=X\setminus U$ and define $f:U\to \mathbf R$ by $f(x)=\rho(x,A)^{-1}$. Then the map $\phi(x)=(x,f(x))$ is continuous from $(U,\rho)$ to $(X\times\mathbf R,\delta)$ where $\delta$ is the obvious complete product metric. It suffices to see that the range of $\phi$ is closed.
    3. If $E$ is a subset of a metric space $(X,\rho)$, then its boundary, $\partial E$ is closed. Show that if $x\in E\setminus \partial E$, then $x\in \operatorname{Int}(E)$. If $E$ is closed, show that $\partial E$ has no interior.
    4. Suppose that $X$ is a complete metric space without isolated points, and that $O_n$ is open and dense for all $n\ge1$. Show that $\bigcap_{n=1}^\infty O_n$ is uncountable. (Note that if $x$ is not isolated, then $X\setminus \{x\}$ is open and dense.)
    5. Suppose that $(X,\rho)$ is a compact metric space and that $f:X\to X$ satisfies $\rho(f(x),f(y))<\rho(x,y)$ whenever $x\not=y$. Show that $f$ has a unique fixed point.
    6. Let $f(x)=\pi/2+x-\arctan(x)$. Show that $|f(x)-f(y)|<|x-y|$ for all $x,y\in\mathbf{R}$ with $x\not=y$, but that $f$ does not have a fixed point. Compare this result to the previous problem.
Monday, October 5:
  • Homework: Turn in problems 11-23 on Wednesday, October 7th via Gradescope. It is unlikely that gradescope will know about homework #2 prior to Wednesday morning.
  • Work:
    1. Show that the countable union of sets of measure zero in $\mathbf{R}$ has measure zero. (I am using the term "measure zero" instead of "content zero" as that will be more relvant down the road.)
    2. Suppose $f:[a,b]\to\mathbf{R}$ is bounded, and let $\mathcal P$ and $\mathcal Q$ be partitions of $[a,b]$. Prove that $L(f,\mathcal P)\le U(f,\mathcal Q)$, where $L(f,\mathcal P)$ and $U(f,\mathcal Q)$ are the lower and upper Riemann sums, respectively, for $f$ on $[a,b]$. (Hint: the result is trivial if $\mathcal P=\mathcal Q$; now let $\mathcal R=\mathcal P\cup\mathcal Q$.)
    3. Prove that a bounded function $f:[a,b]\to\mathbf{R}$ is Riemann integrable on $[a,b]$ if and only if for all $\epsilon>0$ there is a partition $\mathcal P$ of $[a,b]$ such that $$ U(f,\mathcal P)-L(f,\mathcal P)<\epsilon. $$
Wednesday, October 7:
  • Work:
    1. Suppose that $(X,\mathcal M)$ is a measurable space. Show that if $\mathcal M$ is countable, then $\mathcal M$ is finite. (Hint: since $\mathcal M$ is countable, you can show that $\omega_{x}=\bigcap\{\,E:\text{$E\in\mathcal M$ and $x\in E$}\,\}$ belongs to $\mathcal M$. The sets $\{\omega_{x}\}_{x\in X}$ partition $X$.) This is from Rudin's Real & Complex, page 31, #1
    2. Let $X$ be an uncountable set and let $\mathcal M$ be the collection of subsets $E$ of $X$ such that either $E$ or $E^{C}$ is countable. Prove that $\mathcal M$ is a $\sigma$-algebra.
    3. Let $(a_{n})$ be a sequence in $[-\infty,\infty]$.
      1. Show that $\liminf_{n}a_{n}\le \limsup_{n}a_{n}$.
      2. Suppose that $\lim a_{n}$ exists and equals $L\in [-\infty,\infty]$. Show that $\limsup_{n}a_{n}=L=\liminf_{n}a_{n}$.
      3. Suppose that $\limsup_{n}a_{n}=L=\liminf_{n}a_{n}$. Show that $\lim_{n}a_{n}$ exists and equals $L$.
  • Borel Sets (OPIONAL -- Do not turn in):
  • If you are willing to learn about ordinals and transfinite induction (Secton 0 of Folland's Real Analysis is a good source for this), then you can see that the cardinality of the Borel sets in $\mathbf R$ is the same as the same of that of $\mathbf R$ itself (usually written $\mathfrak c$). In particular, $\mathcal B(\mathbf R)$ is strictly smaller than $\mathcal P(\mathbf R)$. Moreover, you get some insight as to what is in the $\sigma$-algebra generated by a subset of $\mathcal P(\mathbf R)$. I wrote some directed problems about this some years ago. If you prefer, here are the solutions.
Friday, October 9:
  • Homework Solutions: Here are selected solutions for the homework. (Last modified December 31, 1969)
  • Work:
    1. Recall from calculus that if $\{a_{n}\}$ is a sequence of nonnegative real numbers, then $\sum_{n=1}^{\infty} a_{n} = \sup_{n} s_{n}$, where $s_{n} = a_{1}+\dots+ a_{n}$. (Note the value $\infty$ is allowed.)
      1. Show that $\sum_{n=1}^{\infty} a_{n} =\sup\{\,\sum_{k\in F}a_{k}: \text{$F$ is a finite subset of $\mathbf{N}=1,2,3,\dots$}\,\}$. (The point of this problem is that if $I$ is a (not necessarily countable) set, and if $a_{i}\ge0$ for all $i\in I$, then we can define $\sum_{i\in I} a_{i} = \sup\{\,\sum_{k\in F}a_{k}: \text{$F$ is a finite subset of $I$}\,\}$, and our new definition coincides with the usual one when both make sense.)
      2. Now let $X$ be a set and $f:X\to[0,\infty)$ a function. For each $E\subset X$, define \begin{equation*} \nu(E) := \sum_{x\in E} f(x). \end{equation*} Show that $\nu$ is a measure on $\bigl(X,\mathcal P(X)\bigr)$. (Note that some care is required here. We can't pass operations like supremums and limits though infinite sums without justification! Also note that in lecture, we considered the special cases of counting measure, where $f(x)=1$ for all $x\in X$, and the delta measure at $x_{0}$, where $f(x_{0})=1$ for some $x_{0}\in X$ and $f(x)=0$ otherwise. Another important example is the case where $\sum_{x\in X}f(x)=1$. Then $f$ is a (discrete) probability distribution on $X$ and $\nu(E)$ is the probability of the event $E$ for this distribution.)
      3. Let $X$, $f$, and $\nu$ be as in part (2). Show that if $\nu(E)<\infty$, then $\{\,x\in E:f(x)>0\,\}$ is countable. Hint: if $\{\,x\in E:f(x)>0\,\}$ is uncountable, then for some $m\in \mathbf{N}$, the set $\{\,x\in E:f(x)>\frac1m\,\}$ is infinite. (Note that this last result says that discrete probability distributions ``live on'' countable sample spaces.)
Monday, October 12:
  • Work:
    1. Suppose that $f,g:(X,\mathcal M)\to[-\infty,\infty]$ are measurable functions. Prove that the sets $$ \{\,x:f(x)< g(x)\,\}\quad\text{and}\quad\{\,x:f(x)=g(x)\,\}$$ are measurable. (Remark: if $h=f-g$ were defined, then this problem would be much easier (why?). The problem is that $\infty-\infty$ and $-\infty+\infty$ make no sense, so $h$ may not be everywhere defined.)
Friday, October 16:
  • A Cheat: To construct some counterexamples below, let's assume that we have defined Lebesgue $m$ on $(\mathbf R,\mathcal B(\mathcal R))$ so that the Lebesgue integral conicides with the Riemann integral if $f$ is Riemann integrable. Thus you can assume $1_{[-n,n]}$ has integral $2n$ or that $1_{[0,\infty)}$ has infinite integral.
  • Work:
    1. Let $X$ be an uncountable set and $\mathcal M$ the $\sigma$-algebra from problem #28 above. Define $\mu:\mathcal M\to [0,\infty]$ by $\mu(E)=0$ if $E$ is countable and $\mu(E)=1$ if $E$ is uncountable.
      1. Show that $\mu$ is a measure on $(X,\mathcal M)$.
      2. Describe the measurable functions $f:X\to\mathbf C$ and their integrals. (Hint: show that a measurable function must be constant off a countable set; that is, $f$ must be constant $\mu$-almost everywhere.)
    2. Suppose that $f_n:X\to\mathbf R$ is measurable for all $n\in \mathbf N$.
      1. Show that it need not be the case that $$\int_X \liminf_n f_n \,d\mu\le \liminf_n \int_X f_n\,d\mu.$$ That is, the conclusion of Fatou's Lemma is false if we no not assume the $f_n$ are non-negative.
      2. Suppose now that $f_n\le f_{n+1}$ for each $n\in\mathbf N$. Show that it need not be the case that $$\lim_n\int_X f_n\,d\mu=\int_X f\,d\mu.$$ That is, the conclusion of the Monotone Convergence Theorem can fail if we don't insist that the $f_n$ are non-negative. What if we assume $f_1\in\mathcal L^1(X)$?
    3. Suppose that $(X,\mathcal M,\mu)$ is a measure space with $\mu(X)<\infty$. (We say that $(X,\mathcal M,\mu)$ is a finite measure space.) Show that if $f_n:X\to \mathbf C$ is a sequence of bounded measurable functions converging uniformly to $f$, then $f\in \mathcal L^1(X)$ and $$\lim_n\int_X f_n\,d\mu=\int_Xf\,d\mu.$$ Show that the hypothesis that $\mu(X)<\infty$ can't be ommited.
    4. Suppse that $f\in \mathcal L^1(X,\mathcal M,\mu)$. Show that for all $\epsilon >0$ there is a $\delta>0$ such that $\mu(E)<\delta$ implies that $$\int_E|f|\,d\mu<\epsilon.$$ (Hint: First work the problem assuming that $f$ is bounded. Then let $E_n =\{\, x:|f(x)|\le n\,\}$ and $f_n=1_{E_n}\cdot f$. Then observe that $\int_X|f-f_n|\,d\mu\to 0$.)
Monday, October 19:
  • Work:
    1. Suppose that $\{A_n\}_{n=1}^\infty$ are subsets of a set $X$. Let $B_1=A_1$ and $B_n=A_n\setminus \bigcup_{k=1}^{n-1}A_k$ for $n\ge2$. Show that the $B_n$ are pairwise disjoint, $B_n \subset A_n$, and that $\bigcup_{k=1}^nA_k=\bigcup_{k=1}^n B_k$ for all $n$. Observe that if $(X,\mathcal M)$ is a measurable space and the $A_n$ are measurable, then so are the $B_n$. (I sometimes call this process `disjointification'. This really a joke since I don't believe `disjointification' is really a word.)
Wednesday, October 21:
  • Work:
    1. Suppose that $Y$ is a topological space and that $\mathcal M$ is a $\sigma$-algebra in $Y$ containing all the Borel sets. Suppose that $\mu$ is a measure on $(Y,\mathcal M)$ such that for all $E\in\mathcal M$ $$ \mu(E)=\inf\{\,\mu(V):\text{$V$ is open and $E\subset V$}\,\}. $$ Suppose also that $$Y=\bigcup_{n=1}^\infty Y_n\quad\text{with $\mu(Y_n)<\infty$ for all $n\ge1$.}\label{sfin}\tag{$*$}$$ In this case we say that $\mu$ is a $\sigma$-finite outer-regular measure on $(Y,\mathcal M)$.
      1. Show that Lebesgue measure $m$ on $(\mathbf R,\mathcal L)$ is an example of a $\sigma$-finite outer regular measure.
      2. If $E\in\mathcal M$ and if $\epsilon>0$, then show that there is an open set $V$ and a closed set $F$ such that $F\subset E\subset V$ with $\mu(V\setminus F)<\epsilon$. (Hint: first assume $\mu(E)<\infty$. Then use \eqref{sfin}.)
      3. Recall that a countable intesection of open sets is called a $G_\delta$-set, and that a countable union of closed sets is called a $F_\sigma$-set. Show that if $E\in \mathcal M$, then there is a $G_\delta$-set $G$ and a $F_\sigma$-set $A$ such that $A\subset E\subset G$ and $\mu(G\setminus F)=0$.
      4. Use the above to conclude that $(\mathbf R,\mathcal L,m)$ is the completion of restriction of Lebesgue measure to the Borel subsets of $\mathbf R$.
    2. We define the symmetric difference of two subsets $E$ and $F$ to be $E\Delta F:=(E\setminus F)\cup (F\setminus E)$. In this problem, you may use without proof that every open subset of $\mathbf R$ is a countable disjoint union of open intervals. Suppose that $E\subset \mathbf R$ is a set of finite Lebesgue measure. Let $\epsilon>0$. Show that there is finite disjoint union $F$ of open intervals such that $m(E\Delta F)<\epsilon$.
    3. Let $(X,\mathcal M,\mu)$ be a measure space and let $(X,\mathcal M_0,\mu_0)$ be its completion.
      1. If $f:X\to\mathbf C$ is $\mu_0$-measurable, show that there is a $\mu$-measurable function $g:X\to\mathbf C$ such that $f=g$ for $\mu_0$-almost all $x$. (Hint: show that it suffices to assume that $f$ is a $\mu_0$-measurable simple function.)
      2. Further observe that there is a $\mu$-null set $N$ such that $f(x)=g(x)$ if $x\notin N$.
      3. What does this result say about Lebesgue measurable functions on $\mathbf R$ and Borel functions? (See the last part of the question #37.)
Monday, October 26:
  • Homework Solutions: Here are selected solutions for the homework. (Last modified December 31, 1969)
  • Work:
    1. Show that a Hahn decomposition for a real-valued measure $\nu$ is unique up to null sets as claimed in lecture. Also verify that the singular measures in the Jordan decomposition of a real-valued measure $\nu$ are unique.
Wednesday, October 28:
  • Work:
    1. Suppose that $(X,\mathcal M,\mu)$ is a measure space and that $f:X\to[0,\infty]$ is measurable. Let $\nu$ be the measure on $(X,\mathcal M)$ defined by $$\nu(E)=\int_E f(x)\,d\mu(x)\quad\text{for all $E\in\mathcal M$.}$$ Show that if $g:X\to\mathbf C$ is measurable, then $g\in \mathcal L^1(X,\nu)$ if and only if $fg\in \mathcal L^1(X,\mu)$ and that $$\int_X g(x)\,d\nu(x)=\int_X g(x)f(x)\,d\mu(x). $$ (Hint: start with the corresponding result from Lecture 14.)
    2. Let $\mu$, $\nu$, and $\lambda$ be $\sigma$-finite measures on $(X,\mathcal M)$. We'll denote the Radon-Nikodym derivative of $\nu$ by $\mu$ by $\displaystyle{ \frac{d\nu}{d\mu}}$.
      1. Show that $\displaystyle{ \frac{d\nu}{d\mu}}$ is determined $\mu$-almost everywhere.
      2. Suppose that $\nu\ll\mu\ll\lambda$. Show that $\displaystyle{{\frac{d\nu}{d\lambda}}= \frac{d\nu}{d\mu}\frac{d\mu}{ d\lambda}}$. Of course, "$=$" means "equal almost everywhere $[\lambda]$."
      3. Suppose that $\mu\ll\nu$ and $\nu\ll\mu$ (we say the $\mu$ and $\nu$ are equivalent and write $\nu\approx\mu$). Show that $\displaystyle{\frac{d\mu}{d\nu}= \left[\frac{d\nu}{d\mu}\right]^{-1}}$. Again "$=$" means "equal almost everywhere $[\mu]$ (or $[\nu]$)".
    3. Show that the $\sigma$-finite hypothesis is necessary in the Radon-Nikodym theorem. (Hint: let $\nu$ be Lebesgue measure on $[0,1]$ and let $\mu$ be counting measure (restricted to the Lebesgue measurable sets in $[0,1]$).)
    4. Suppose that $\rho$ is a premeasure on an algebra $\mathcal A$ of sets in $X$.
      1. Show that $$\rho^*(E)=\inf\{\sum_{k=1}^\infty \rho(A_k):\text{each $A_k\in\mathcal A$ and $E\subset \bigcup A_k$}\}$$ is an outer measure on $X$.
      2. Show that $\rho^*(A)=\rho(A)$ for all $A\in\mathcal A$.
      3. Show that each $A\in \mathcal A$ is $\rho^*$-measurable.
Friday, October 30:
  • Cantor-Lebesgue Function: At one point, several of you expressed interest in the relationship between the Cantor set and ternary expansions. This leads to the Cantor-Lebesgue function which has some interesting properties. It can also be used to give another proof that there are Borel subsets of $\mathbf R$ that are not Lebesgue measurable. Follow this link for a discussion of this. Doing so is entirely optional and this material is not officially part of the course.
  • Homework:Problems 36-45 will be due Friday, November 6 via gradescope.
  • Work:
    1. Show that the conlusion of Egoroff's Theorem can fail if $\mu(X)=\infty$, but is still valid if "$\mu(X)<\infty$" is replaced by "$|f_n(x)|\le g(x)$ with $g\in\mathcal L^1(\mu)$". (For the statement of Egoroff's Theorem and the proof in the finite measure case, see Lecture 18.)
Monbday, November 2:
  • Homework:Problems 36-45 will be due Friday, November 6 via gradescope.
  • Work:
    1. Let $\mathcal B(\mathbf R)$ be the Borel $\sigma$-algebra in $\mathbf R$ and $\mathcal B(\mathbf R^2)$ the Borel $\sigma$-algebra in $\mathbf R^2$. Show that $\mathcal B(\mathbf R)\otimes \mathcal B(\mathbf R)=\mathcal B(\mathbf R^2)$. (You may use the observation that every open set in $\mathbf R^2$ is a countable union of open rectangles.)
    2. Let $(X,\mathcal M,\mu)$ and $(Y,\mathcal N,\nu)$ be complete $\sigma$-finite measure spaces. Let $(X\times Y,\mathcal L,\lambda)$ be the completion of $(X\times Y,\mathcal M\otimes \mathcal N,\mu{\times}\nu)$.
      1. Suppose that $E\in \mathcal M\otimes \mathcal N$ and $\mu{\times}\nu(E)=0$. Show that $\mu(E^y)=0=\nu(E_x)$ for $\mu$-almost all $x$ and $\nu$-almost all $y$.
      2. Suppose that $f$ is $\mathcal L$-measurable and that $f(x,y)=0$ for $\lambda$-almost all $(x,y)$. Show that there is a $\mu$-null set $M$ and a $\nu$-null set $M$ such that for all $x\notin M$ and $y\notin N$, $f_x$ and $f^y$ are integrable and that $$\int_X f^y\,d\mu =0=\int_Y f_x\,d\nu.$$
Friday, November 6:
    1. Let $\mathcal B$ be the Borel $\sigma$-algebra in $[0,1]$. Let $m$ be Lebesgue measure on $([0,1],\mathcal B)$ and let $\nu$ be counting measure on $([0,1],\mathcal B)$. Let $D=\{\,(x,x)\in [0,1]\times[0,1]:x\in [0,1]\,\}$ be the diagonal. Compare the integrals \begin{equation} \label{eq:40} \int_{[0,1]\times[0,1]}1_D\,d (m{\times}\nu),\quad \int_{[0,1]}\int_{[0,1]}1_D\,dm\,d\nu,\quad\text{and} \quad\int_{[0,1]}\int_{[0,1]}1_D\,d\nu\,dm, \end{equation} and comment on the relationship to the Fubini-Tonelli Theorems. (You will have to resort to the definition of $m\times\nu$ to compute the first integral.)
    2. Let $(X,\mathcal M,\mu)$ and $(Y,\mathcal M,\nu)$ both be counting measure $(\mathbf N,\mathcal P(\mathbf N),\nu)$. Define $f:X\times Y\to \mathbf R$ by \begin{equation} \label{eq:46} f(m,n)= \begin{cases} \phantom{-}1&\text{if $m=n$,}\\-1&\text{if $m=n+1$, and}\\\phantom{-}0&\text{otherwise.} \end{cases} \end{equation} Observe that $f$ is not integrable and that the two iterated integrals $\int_{Y}\int_{X}f\,d\mu\,d\nu$ and $\int_{X}\int_{Y}f\,d\nu\,d\mu$ are not equal. What is the moral here?
    3. Let $(X,\mathcal M,\mu)$ be a measusre space.
      1. Show that $\|\cdot\|_\infty$ is a norm on $L^\infty(X)$.
      2. Show that $f_n\to f$ in $L^\infty(X)$ if and only if there is a null set $E\in\mathcal M$ such that $f_n\to f$ uniformly on $X\setminus E$.
      3. Show that $L^\infty(X)$ is a Banach space.
      4. Show that (measurable) simple functions are dense in $L^\infty(X)$. (Hint: re-read our result on MNNSFs from Lecture 12.)
Monday, November 9:
  • Homework Solutions: Here are selected solutions for the homework. (Last modified December 31, 1969)
  • Work:
    1. [OPTIONAL -- DO NOT TURN IN] In the notation of our technical proposition from Lecture 25, show that if $\mu$ is semifinite, $q<\infty$, and $M_q(g)<\infty$, then $\{\,x:|g(x)|>\epsilon\,\}$ has finite measure for all $\epsilon>0$. Hence $S_g$ is $\sigma$-finite as required in the proof.
Wednesday, November 11:
    1. Let $(X,\mathcal M,\mu)$ be a measures space with $\mu(X)<\infty$. If $1\le r < s\le \infty$, show that $\mathcal L^s(X)\subset \mathcal L^r(X)$. (Hint: apply Holder to $|f|^r=1\cdot |f|^r$ where $1$ denotes the constant function.)
    2. Show that if $1\le r< s \le \infty$, then $\ell^r\subset \ell^s$.
    3. Let $(X,\mathcal M,\mu)$ be a measures space. Suppose that $1\le r< t < s\le \infty$. Show that $\mathcal L^r(X)\cap \mathcal L^s(X)\subset \mathcal L^t(X)$.
Friday, November 20:
  • Homework Solutions: Here are selected solutions for the homework. (Last modified December 31, 1969)


Dana P. Williams
Last updated November 20, 2020 12:44:48 EST